Sei sulla pagina 1di 93

Engineering

Mechanics 2 nd
Edition
By Ferdinand Singer

Solution Manual
Problem 236 Thus, R = 110 lb downward at 6 ft to the right of
A parallel force system acts on the lever A. answer
shown in Fig. P-236. Determine the
magnitude and position of the resultant. Problem 237
Determine the resultant of the four parallel forces
acting on the rocker arm of Fig. P-237.

Solution 236

R=F
Solution 237
R=30+6020+40
R=F
R=110 lb downward

R=504020+60

R=50 lb downward

MO=Fd

MA=xF MO=50(6)+40(2)20(3)+60(8)

MA=2(30)+5(60)7(20)+11(40) MO=200 lbft clockwise

MA=660 ftlb clockwise


Rd=MO

Rd=MA 50d=200

110d=660 d=4 ft to the right of O

d=6 ft to the right of A


Thus, R = 50 lb downward at 4 ft to the right of 3330d=3(1320)+4(2010)
point O. answer
3330d=12000

d=3.6 m
Problem 238
The beam AB in Fig. P-238 supports a
load which varies an intensity of 220 Thus, R = 3330 N downward at 3.6 m to the left of
N/m to 890 N/m. Calculate the A. answer
magnitude and position of the resultant
load.
Problem 239
The 16-ft wing of an airplane is subjected
to a lift which varies from zero at the tip
to 360 lb per ft at the fuselage according
to w = 90x1/2 lb per ft where x is
measured from the tip. Compute the
resultant and its location from the wing
tip.

Solution 238
Solution 239
F1=6(220)=1320 N

F2=12(6)(670)=2010 N

R=160ydx=90160x1/2dx
R=F1+F2=1320+2010
R=90[x3/2/{3/2}] |160=60[x3/2] |160
R=3330 N
R=60(163/203/2)

R=3840 lb upward
Rd=3F1+4F2
Rd=160 x(ydx)=90160 x(x1/2dx) of 4-in. diameter has been cut in the
plate. Locate the center of gravity the
3840d=90160 x3/2dx plate. Hint: The weight of the plate is
equivalent to the weight of the original
3840d=90[x5/2/{5/2}]160=36[x5/2]160
plate minus the weight of material cut
3840d=36(165/205/2) away. Represent the original plate weight
of plate by a downward force acting at
3840d=36864 the center of the 10 14 in. rectangle.
Represent the weight of the material cut
d=9.6 ft away by an upward force acting at the
center of the circle. Locate the position of
the resultant of these two forces with
Thus, R = 3840 lb upward at 9.6 ft from the tip of
respect to the left edge and bottom of
the wing. answer
the plate.
Another Solution

Solution 240

R= area of parabola

R=2/3bh=2/3(16)(360)

R=3840 lb okay!

x= centroid of parabola
W=(14)(10)
x=3/5h=3/5(16)
W=140
x=9.6 ft okay!

F=[14(42)]

F=4
Problem 240
The shaded area in Fig P-240 represents
a steel plate of uniform thickness. A hole R=WF=1404
R=(1404)

Rx=7W9F

(1404)x=7(140)9(4)

(1404)x=7(140)9(4)

(1404)x=98036

x=980361404

x=6.8 in.
Solution 241

Ry=5W6F sin30=a/3

(1404)y=5(140)6(4) a=3sin30

(1404)y=5(140)6(4) a=1.5 m

(1404)y=70024

y=700241404

y=4.9 in.

Thus, the centroid is located at 6.8 in. to the right of


left edge and 4.9 in. above the bottom
edge. Answer

Problem 241 cos60=b/a


Locate the amount and position of the
resultant of the loads acting on the Fink b=acos60
truss in Fig. P-241.
b=1.5cos60

b=0.75 m
Magnitude of resultant
R=Fv

R=2(890)+2(1335)+1780+8900

R=15130 N downward

Sum of vertical forces


Location of resultant R=Fv
Rd=Fx
300=100+PF+200
Rd=1335(3b)+8900(3)+1780(4.5)+1335(6+b)+890
(9) P=200+F

15130d=1335(30.75)+8900(3)+1780(4.5)+1335(6
+0.75)+890(9) Moment about point A
Rd=Fx
15130d=1335(2.25)+8900(3)+1780(4.5)+1335(6.75
)+890(9) 300(4)=100(0)+P(2)F(5)+200(7)

15130d=54735 1200=2P5F+1400

d=3.62 m to the right of A 2P5F+200=0

2(200+F)5F+200=0
Thus, R = 15 130 N downward at 3.62 m to the
right of left support. answer 400+2F5F+200=0

600=3F

Problem 242 F=200 lb answer


Find the value of P and F so that the four
forces shown in Fig. P-242 produce an
P=200+200
upward resultant of 300 lb acting at 4 ft
from the left end of the bar.
P=400 lb answer

Problem 243
The resultant of three parallel loads (one
is missing in Fig. P-243) is 13.6 kg acting
up at 3 m to the right of A. Compute the
magnitude and position of the missing
Solution 242
load. and MO = 360 kNm counterclockwise.
Determine the point at which the
resultant intersects the x-axis.

Solution 260

Solution 243

Sum of vertical forces


R=Fv MO=360 kNm

60a=360
13.6=18F+27
a=6 m
F=31.4 kg downward
The x-intercept is at 6 m to the left of the
origin. answer

Problem 261
In a certain non-concurrent force system
it is found that Fx = -80 lb, Fy = +160
lb, and MO = 480 lbft in a
counterclockwise sense. Determine the
point at which the resultant intersects
Moment about point A the y-axis.
Rd=Fx
Solution 261
13.6(3)=18(0.6)31.4x+27(4)
MO=480 lbft
x=2.48 m
80b=480

Thus, F = 31.4 kg downward at 2.48 m to the right b=6 ft


of left support. answer
The y-intercept of
Problem 260 the resultant is 6 ft
The effect of a certain non-concurrent above the
force system is defined by the following origin. answer
data:Fx = +90 kN, Fy = -60 kN,
=44.21
Problem 262
Determine completely the resultant of
Maxle=Mcenter
the forces acting on the step pulley
shown in Fig. P-262. Maxle=250(1.25)+1250(0.5)750(1.25)

Maxle=0

Thus, R = 1254.89 lb downward to the right at x =


44.21 and passes through the axle.

Problem 263
Determine the resultant of the force
system shown in Fig. P-263 and its x and
Solution 262
y intercepts.
Rx=Fx

Rx=750sin60+250

Rx=899.52 lb to the right

Ry=Fy

Ry=750cos601250

Ry=875 lb

Ry=875 lb downward

Solution 263
2 2
R=Rx +Ry
Rx=Fx
R=899.522+8752
Rx=300sin30224(2/5)+361(2/13)
R=1254.89 lb
Rx=149.895 lb to the right
tanx=Ry/Rx

tanx=875899.52
Rya=MO

59.613a=100.598
Ry=Fy
a=1.688 ft to the right of the origin
Ry=300cos30+224(1/5)361(3/13)
Rxb=MO
Ry=59.613 lb upward
149.895b=100.598
R=Rx2+Ry2
b=0.671 ft below the origin
R=149.8952+59.6132
Thus, R = 161.314 lb upward to the right at x =
R=161.314 lb 21.69 and intercepts at (1.668, 0) and (0, -0.671).

tanx=Ry/Rx
Problem 264
Completely determine the resultant with
tanx=59.613/149.895
respect to point O of the force system
x=21.69 shown in Fig. P-264.

MO=M

MO=(300sin30)(2)+224(15)(2)+361(
213)(1)

MO=100.598 lbft counterclockwise


Solution 264 MO=141.4(1/2)(3)(300sin60)(4)(3
00cos60)(4)260(12/13)(1)+260(5/13)(4)
Rx=Fx

Rx=141.4(1/2)+300sin60+260(12/13) MO=1779.18 Nm
240sin30 MO=1779.18 Nm clockwise
Rx=479.79 N to the right

Rxb=MO

479.79b=1779.18
Ry=Fy b=3.71 m above point O
Ry=141.4(1/2)300cos60+260(5/13)+2
40cos30 Rya=MO

Ry=257.83 N upward 257.83a=1779.18

a=6.9 m to the left of point O


R=Rx +Ry
2 2

Thus, R = 544.68 N upward to the right at x =


R=479.792+257.832 28.25. The intercepts of R are (-6.9, 0) and (0,
3.71).
R=544.68 N

tanx=Ry/Rx Problem 265


Compute the resultant of the three forces
tanx=257.83/479.79 shown in Fig. P-265. Locate its
intersection with X and Y axes.
=28.25

MO=Fd
R=Rx2+Ry2

R=810.742+510.302

R=957.97 lb

tanx=Ry/Rx

tanx=510.30/810.74

x=32.19

MO=Fd
Solution 265
MO=390(12/13)(3)+390(5/13)(5)722(2/
Rx=Fx
13)(4)+(300sin30)(3)

Rx=390(12/13)+722(3/13)300sin30
MO=1481.97 lbft
Rx=810.74 lb to the right
MO=1481.97 lbft clockwise

Rxb=MO

Ry=Fy 810.74b=1481.97

Ry=390(5/13)722(2/13)300cos30 b=1.83ft above point O

Ry=510.30 lb
Rya=MO
Ry=510.30 lb downward
510.30a=1481.97
a=2.90ft to the right of point O R=4803.852+270002

R=27424.02 lb
Thus, R = 957.97 lb downward to the right at x =
32.19. The x-intercept is at 2.90 ft to the right of O
and the y-intercept is 1.83 ft above point O.
tanx=Ry/Rx

tanx=27000/4803.85
Problem 266
Determine the resultant of the three x=79.91
forces acting on the dam shown in Fig. P-
266 and locate its intersection with the
base AB. For good design, this Righting moment
intersection should occur within the RM=24000(187)+6000(4)
middle third of the base. Does it?
RM=288000 lbft

Overturning moment
OM=10000(6)

OM=60000 lbft

Moment at the toe (downstream side - point B)


MB=RMOM

MB=28800060000
Solution 266
MB=228000 lbft counterclockwise
Rx=Fx

Rx=100006000sin60

Rx=4803.85 lb to the right

Ry=Fy

Ry=24000+6000cos60

Ry=27000 lb downward
Location of Ry as measured from the toe
R=Rx2+Ry2 Ryx=MB
27000x=228000 Ry=Fy

x=8.44 ft to the left of B (within the Ry=(1120+2240+1120)(2/5)+3000+20


middle third) 00+1000

Thus, R = 27 424.02 lb downward to the right at x Ry=10007.03 N downward


= 79.91 and passes through the base at 8.44 ft to
the left of B which is within the middle third.
R=Rx2+Ry2
Problem 267
The Howe roof truss shown in Fig. P-267 R=4003.522+10007.032
carries the given loads. The wind loads
are perpendicular to the inclined R=10778.16 N
members. Determine the magnitude of
the resultant, its inclination with the
horizontal, and where it intersects AB. tanx=Ry/Rx

tanx=10007.03/4003.52

x=68.2

Solution 267

Rx=Fx

Rx=(1120+2240+1120)(1/5)+2000
MA=Fd
Rx=4003.52 N to the right
MA=2240(3.354)+1120(3.354)(2)+200
0(1.5)+3000(3)+2000(6)+1000(9)

MA=48026.37 Nm clockwise

Ryx=MA

10007.03x=48026.37

x=4.8 m to the right of A


Thus, R = 10 778.16 N downward to the right at x Thus, F4=200 lb to the left
= 68.2 passing 4.8 m to the right of A.

Problem 268
The resultant of four forces, of which
three are shown in Fig. P-268, is a
couple of 480 lbft clockwise in sense. If
each square is 1 ft on a side, determine
the fourth force completely.

Assuming F4 is above point O


MO=C

F4d110(4)120(2)=480

200d110(4)120(2)=480

d=5.8 ft
Solution 268

Let F4 = the fourth force and for couple resultant, R


is zero. d is positive, thus, the assumption is correct that F4
is above point O.

Rx=0 Therefore, the fourth force is 200 lb acting


horizontally to the left at 5.8 ft above point
110+150(3/5)+F4x=0 O. answer

F4x=200 lb

F4x=200 lb to the left


Problem 269
Repeat Prob. 268 is the resultant is 390
Ry=0 lb directed down to the right at a slope of
5 to 12 passing through point A. Also
150(4/5)120+F4y=0 determine the x and y intercepts of the
missing force F.
F4y=0
Fy=Ry

150(4/5)+120+F4y=150
Solution 269

Let F4 = the fourth force


F4y=150 lb downward

Rx=390(12/13) F4=F4x2+F4y2

Rx=360 lb to the right F4=1602+1502

F4=219.32 lb
Fx=Rx

110+150(3/5)+F4x=360 tanx=F4y/F4x

F4x=160 lb to the right tanx=150/160

x=43.15
Ry=390(5/13)

Ry=150 lb downward MO=2Rx+3Ry

MO=2(360)+3(150)

MO=1170 lbft clockwise

Resolve F4 into components at the x-axis


Fd=MO
110(4) + 120(2) + F_{4y)i_x = 1170
110(4)+120(2)+150ix=1170 For horizontal resultant, Ry = 0 and Rx = R

ix=3.27 ft to the right of O


MR=MB

Resolve F4 into components at the y-axis R(1)=Fx(2)+Fy(1)


Fd=MO
R(1)=316(1/10)(2)+316(3/10)(1)
110(4)+120(2)+F4xiy=1170
R=499.64 lb to the right at A
110(4)+120(2)+160iy=1170

iy=3.06 ft above point O

Thus, F4 = 219.32 lb downward to the right at x =


43.15 with x-intercept ix = 3.27 to the right of O,
and y-intercept iy = 3.06 ft above point O.

Problem 270
The three forces shown in Fig. P-270 are
required to cause a horizontal resultant
acting through point A. If F = 316 lb,
determine the values of P and T. Hint:
Apply MR = MB to determine R, then
MR = MC to find P, and finally
MR = MD or Ry = Y to compute T. MC=MR

Fx(4)Fy(2)+Py(4)=R(3)

316(1/10)(4)316(3/10)(2)+P(2/5)(4)=
499.64(3)

P=474.82 lb answer

MD=MR

Fx(4)+Fy(2)Ty(4)=R(3)

316(1/10)(4)+316(3/10)(2)T(2/13)(4)=
499.64(3)

Solution 270 T=225.18 lb answer


Problem 271
The three forces in Fig. P-270 create a
vertical resultant acting through point A.
If T is known to be 361 lb, compute the
values of F and P.

MB=MR

Fx(2)+Fy(1)=R(1)

F(1/10)(2)+F(3/10)(1)=400.49(1)

F=253.29 lb answer

Solution 271
FV=R

For vertical resultant, Rx = 0 and Ry = R Py+TyFy=R

P(2/5)+T(2/13)F(2/10)=R
MR=ME
P(2/5)+361(2/13)253.29(2/10)=400.4
R(2)=Tx(4)Ty(2) 9
R(2)=361(3/13)(4)361(2/13)(2) P=507.06 lb answer

R=400.49 lb downward at point A


Problem 308
The cable and boom shown in Fig. P-308
support a load of 600 lb. Determine the
tensile force T in the cable and the
compressive for C in the boom.
Fy=0

Tsin75=600sin45

T=439.23 lb (okay!)
Solution 308

FH=0 Fx=0

Ccos45=Tcos30 C=Tcos75+600cos45

C=1.2247T C=439.23cos75+600cos45

C=537.94 lb (okay!)
FV=0

Tsin30+Ccos45=600 Another Solution (By Force Polygon)

Tsin30+(1.2247T)cos45=600

1.366T=600

T=439.24 lb answer

C=1.2247(439.24)

C=537.94 lb answer

Another Solution (By Rotation of Axes)


T/sin45=C/sin60=600/sin75

T=439.23 lb (okay!)
C=537.94 lb (okay!)

FV=0

Psin25+Ncos55=400
Problem 309
(0.9038N)sin25+Ncos55=400
A cylinder weighing 400 lb is held against
a smooth incline by means of the
0.9556N=400
weightless rod AB in Fig. P-309.
Determine the forces P and N exerted on N=418.60 lb answer
the cylinder by the rod and the incline.

P=0.9038(418.60)

P=378.34 lb answer

Another Solution (By Rotation of Axes)

Fx=0

Pcos30=400sin55

P=378.35 lb (ok!)
Solution 309

FH=0

Pcos25=Nsin55

P=0.9038N

Fy=0

N=Psin30+400cos55

N=378.35sin30+400cos55
N=418.60 lb (ok!) Solution 310

Another Solution (By Force Polygon)

Fx=0

Pcos=Wsin30
Pcos45=300sin30
P/sin55=N/sin65=400/sin60 P=212.13 lb answer

P=378.35 lb (ok!) Fy=0


N=Psin+Wcos30
N=418.60 lb (ok!) N=212.13sin45+300cos30300cos30
N=409.81 lb answer

Problem 310 Problem 311


A 300-lb box is held at rest on a smooth If the value of P in Fig. P-310 is 180 lb,
plane by a force P inclined at an determine the angle at which it must
angle with the plane as shown in Fig. P-
be inclined with the smooth plane to hold
310. If = 45, determine the value of P 300-lb box in equilibrium.
and the normal pressure N exerted by
the plane.
Solution 311

Fx=0

Pcos=Wsin30
180cos=300sin30
cos=5/6
=33.56 answer

Problem 312
Determine the magnitude of P and F
necessary to keep the concurrent force
system in Fig. P-312 in equilibrium.

Solution 313
Solution 312
Fx=0
FH=0
Fcos60+300=Pcos15+400cos30
Fcos60+200cos45=300+Pcos30
F=1.9318P+92.82
F=317.16+1.7320P

FV=0
Fsin60=200sin45+Psin30
(317.16+1.7320P)sin60=200sin45+Psin30
274.67+1.5P=141.42+0.5
P=133.25 N answer

F=317.16+1.7320(133.25)
F=86.37 N answer
Fy=0
Fsin60+Psin15=200+400sin30
Problem 313
(1.9318P+92.82)sin60+Psin15=200+400sin30
Figure P-313 represents the concurrent 1.6730P+80.38+0.2588P=200+200
force system acting at a joint of a bridge 1.9318P=319.62
truss. Determine the value of P and E to P=165.45 lb answer
maintain equilibrium of the forces.
F=1.9318(165.45)+92.82
F=412.44 lb answer

Problem 314
The five forces shown in Fig. P-314 are in
equilibrium. Compute the values of P and
F.
Solution 315

By Cosine Law
F2=4002+30022(400)(300)cos30
Solution 314
F2=42153.90 lb
Fy=0 F=205.31 lb answer
Fsin30+40cos15=30sin30+20sin60
0.5F=6.3165
F=12.63 kN answer

4002=3002+F22(300F)cos
2(300F)cos=3002+F240022
600(205.31)cos=3002+42153.904002
123186cos=27846.1
cos=0.2260446244
=103.06 answer

The correct position of F would be as shown below.


Fx=0
P+20cos60+40sin15=30cos30+Fcos30
P+10+10.35=25.98+(12.63)(0.8660)
P=5.31 kN answer

Problem 315
The 300-lb force and the 400-lb force
shown in Fig. P-315 are to be held in
equilibrium by a third force F acting at an
unknown angle with the horizontal.
Determine the values of F and . Problem 316
Determine the values of and so that
the forces shown in Fig. P-316 will be in
equilibrium.

Solution 316

By Cosine Law
302=202+4022(20)(40)cos
2(20)(40)cos=202+402302
1600cos=1100
cos=0.6875
=46.57 answer
Solution 317

From the knot where 400-lb load is hanging

FH=0

202=302+4022(30)(40)cos
2(30)(40)cos=302+402202
2400cos=2100
cos=0.875
=28.96 answer
Dsin75=Csin30
Problem 317 D=0.5176C
The system of knotted cords shown in
Fig. P-317 support the indicated weights. FV=0
Compute the tensile force in each cord. Dcos75+Ccos30=400
(0.5176C)cos75+Ccos30=400
C=400 lb answer

D=0.5176(400)
D=207.06 lb answer
At joint B
Fy=0
From the knot where 300-lb load is hanging

FV=0

FABcos30=20sin45
FAB=16.33 kN
Bsin45=300+Ccos30
Fx=0
Bsin45=300+400cos30
FBC=20cos45+FABsin30
B=914.16 lb answer
FBC=20cos45+16.33sin30
FBC=22.31 kN
FH=0
A=Bcos45+Csin30
A=914.16cos45+400sin30
At joint C
A=846.41 lb answer
Fy=0

Problem 318
Three bars, hinged at A and D and
pinned at B and C as shown in Fig. P-
318, form a four-link mechanism.
Determine the value of P that will
prevent motion.
FCDcos15=Psin60
FCD=0.8966P

Fx=0
Pcos60+FCDsin15=FBC
Pcos60+(0.8966P)sin15=22.31
0.7320P=22.31
P=30.48 kN answer

Problem 319
Cords are loop around a small spacer
separating two cylinders each weighing
400 lb and pass, as shown in Fig. P-319
Solution 318 over a frictionless pulleys to weights of
200 lb and 400 lb . Determine the Problem 322
angle and the normal pressure N The Fink truss shown in Fig. P-322 is
between the cylinders and the smooth supported by a roller at A and a hinge at
horizontal surface. B. The given loads are normal to the
inclined member. Determine the
reactions at A and B. Hint: Replace the
loads by their resultant.

Solution 319 Solution 322

FH=0 R=2(1000)+3(2000)
R=8000 lb

400cos=200
cos=0.5 Rx=Rsin30
=60 answer Rx=8000sin30
Rx=4000 lb
FV=0
N+400sin=800 Ry=Rcos30
N+400sin60=800 Ry=8000cos30
N=453.59 lb answer Ry=6928.20 lb

MB=0
60RA=40Ry
60RA=40(6928.20) Bx=30 (okay!)
RA=4618.80 lb answer
From the Force Polygon
MA=0 RA/sin30=RB/sin30=8000/sin120
60BV=20Ry RA=4618.80 lb (okay!)
60BV=20(6958.20) RB=4618.80 lb (okay!)
BV=2309.40 lb
Problem 323
FH=0
The truss shown in Fig. P-323 is
BH=Rx
supported by a hinge at A and a roller at
BH=4000 lb
B. A load of 20 kN is applied at C.
RB=BH2+BV2 Determine the reactions at A and B.
RB=40002+2309.402
RB=4618.80 lb

tanBx=BV/BH
tanBx=2309.40/4000
Bx=30

Thus,
RB = 4618.80 lb at 30 with
horizontal answer

Solution 323
Another Solution
MA=0
tan30=20/y
y=34.64 ft 9RB=(3+1.5)(20cos30)+(9+3)(20sin30)9RB=19R
B=197.94
RB=21.99 kN answer

FH=0
tanBx=y/60 AH=20cos30
tanBx=34.64/60 AH=17.32 kN
MB=0 RA/sin60=RB/sin64.7=RA/sin55.3
9AV=1.5AH+3(20cos30)+3(20sin30) RA=21.07 kN (okay!)
9AV=1.5(17.32)+3(20cos30)+3(20sin30) RB=21.99 kN (okay!)
9AV=107.94
AV=11.99 kN Problem 324
A wheel of 10-in radius carries a load of
RA=AH2+AV2
1000 lb, as shown in Fig. P-324. (a)
RA=17.322+11.992
Determine the horizontal force P applied
RA=21.06 kN
at the center which is necessary to start
tanAx=AVAH the wheel over a 5-in. block. Also find
tanAx=11.9917.32 the reaction at the block. (b) If the force
Ax=34.7 P may be inclined at any angle with the
horizontal, determine the minimum value
Thus, of P to start the wheel over the block;
RA = 21.06 kN down to the left at 34.7 with the the angle P makes with the horizontal;
horizontal. answer and the reaction at the block.
Another Solution

tan30=y/3

y=1.732 m

tanAx=(y+3+1.5)/9
tanAx=(1.732+3+1.5)/9
Ax=34.7 (okay!)

Solution 324

Part (a)
tan30=1000/P
P=1732.05 lb answer

=90Ax=9034.7
=55.3

=18060=18055.360
=64.7

RA/sin60=RA/sin60=20/sin
sin30=1000/R
R=2000 lb answer

Part (b)
P/sin60=1000/sin(30+)
P=1000sin60/sin(30+)

Solution 325

dP/d=(1000sin60cos(30+))/sin2(30+)=0d1
000sin60cos(30+)=0
cos(30+)=0
30+=90
=60 answer

Pmin=1000sin60/sin(30+60)
Pmin=866.02 lb cos=1.5/2
=18060(30+) =41.41
=18060(30+60)
=30 30+=71.41

R/sin=1000/sin(30+) =18.59+
R/sin30=1000/sin(30+60) =90
R=500 lb answer
P/sin71.41=2000/sin
P=2000sin71.41/sin(18.59+)

dP/d={2000sin71.41cos(18.59+)}/sin2(18.59
Problem 325 +)=0
Determine the amount and direction of 2000sin71.41cos(18.59+)=0
the smallest force P required to start the cos(18.59+)=0
wheel in Fig. P-325 over the block. What 18.59+=90
is the reaction at the block? =71.41 answer

Pmin={2000sin71.41}/sin(18.59+71.41)
Pmin=1895.65 lb answer

=18.59+71.41=90
=9071.41=18.59

R/sin=2000/sin
R/sin18.59=2000/sin90
R=637.59 lb answer

Problem 326
The cylinders in Fig. P-326 have the
indicated weights and dimensions.
Assuming smooth contact surfaces, From the FBD of 200 kN cylinder
determine the reactions at A, B, C, and D FV=0
on the cylinders. RCsin=200
RCsin29.93=200
RC=400.85 kN answer

FH=0
RD=RCcos
RD=400.85cos29.93
RD=347.39 kN answer

Solution 326

cos=2.6/(2+1)
=29.93

From the FBD of 400 kN cylinder


FH=0
RA=RCcos
RA=400.85cos29.93
RA=347.39 kN answer

FV=0
RB=400+RCsin FV=0
RB=400+400.85sin29.93 P(2/5)+F(4/5)=18
RB=600 kN answer 5/2P+F=22.5

Substitute F of Equation (1)


Problem 327
5/2P+(5/3P+50)=22.5
Forces P and F acting along the bars
55/6P=27.55
shown in Fig. P-327 maintain equilibrium
P=14.76 kN answer
of pin A. Determine the values of P and
F. From Equation (1)
F=5/3(14.76)+50
F=39 kN answer

Problem 328
Two weightless bars pinned together as
shown in Fig. P-328 support a load of 35
kN. Determine the forces P and F acting
respectively along bars AB and AC that
maintain equilibrium of pin A.

Solution 327

FH=0

Solution 328
F(3/5)=P(1/5)+30
F=5/3P+50 Equation (1)
FH=0 cos=2cos
F(5/41)=P(2/13)
F=0.7104P

FV=0
P(3/13)=F(4/41)+35

P(3/13)=(0.7104P)(4/41)+35
0.3883P=35
P=90.14 kN answer

F=0.7104(90.14)
F=64.03 kN answer From the figure
+=90
=90

Problem 329 Thus,


cos(90)=2cos
Two cylinders A and B, weighing 100 lb
sin=2cos
and 200 lb respectively, are connected
sin/cos=2
by a rigid rod curved parallel to the
tan=2
smooth cylindrical surface shown in Fig.
=63.43 answer
P-329. Determine the angles and that
define the position of equilibrium. =9063.43
=26.57 answer

Problem 332
Determine the reactions for the beam shown in Fig.
P-332.

Solution 329

MO=0
100Rcos=200Rcos
400 lb per ft.

Solution 332
Solution 333
MR2=0
MR4=0
10R1+4(400)=16(300)+9[14(100)]
12R3=4[12(12)(400)]
R1=1580 lb answer
R3=800 lb

MR3=0

12R4=8[12(12)(400)]

R4=1600 lb

MR1=0

10R2+6(300)=14(400)+1[14(100)]

R2=520 lb answer

Problem 333
Determine the reactions R1 and R2 of the beam in
Fig. P-333 loaded with a concentrated load of 1600
lb and a load varying from zero to an intensity of
MR2=0

16R1=13(1600)+12R3

16R1=13(1600)+12(800)

R1=1900 lb answer

MR1=0
MR1=0
16R2=3(1600)+4R3+16R4
7.5R2=1.5(12)+3[3(6)]+6.5[(1/2)(3)(15)]
16R2=3(1600)+4(800)+16(1600)
R2=29.1 kN answer
R2=2100 lb answer

Problem 335
The roof truss in Fig. P-335 is supported by a roller
Problem 334
Determine the reactions for the beam loaded as at A and a hinge at B. Find the values of the
shown in Fig. P-334. reactions.

Solution 334

MR2=0 Solution 335

7.5R1=6(12)+4.5[3(6)]+1[(1/2)(3)(15)] Replace the 3-20 kN forces and 2-10 kN forces by a


single 80 kN force
R1=23.4 kN answer MB=0

15RA=10(60)+7.5(80)+5(50)
RA=96.67 kN answer

MA=0

2RB=12(2000)+6[12(100)]

RB=15600 lb answer

MA=0

15RB=5(60)+7.5(80)+10(50) Problem 337


The upper beam in Fig. P-337 is supported at D and
RB=93.33 kN answer a roller at C which separates the upper and lower
beams. Determine the values of the reactions at A,
B, C, and D. Neglect the weight of the beams.

Problem 336
The cantilever beam shown in Fig. P-336 is built
into a wall 2 ft thick so that it rests against points A
and B. The beam is 12 ft long and weighs 100 lb per
ft.

Solution 337

MC=0
Solution 336
10RD+4(60)=6(190)
MB=0
RD=90 kN answer
2RA=10(2000)+4[12(100)]

RA=12400 lb answer MD=0


10RC=14(60)+4(190)

RC=160 kN answer

Solution 338

From FBD of beam CD


FV=0

RC+R3=P

RC+0.5RC=960

MA=0 RC=640 lb

10RB=4(400)+14(160)
R3=0.5(640)=320 lb answer
RB=384 kN answer

MB=0

10RA+4(160)=6(400)

RA=176 kN answer

Problem 338
The two 12-ft beams shown in Fig. 3-16 are to be
moved horizontally with respect to each other and
load P shifted to a new position on CD so that all
three reactions are equal. How far apart will R2 and
R3 then be? How far will P be from D?
MC=0

12R3=960x

12(320)=960x

x=4 ft

Thus, P is 8 ft to the left of D. answer

From the figure above, Rc is at the midspan of AB


to produce equal reactions R1 and R2. Thus, R2 and
R3 are 6 ft apart. answer

From FBD of beam AB


R1=0.5(640)=320 lb answer
Solution 339
R2=0.5(640)=320 lb answer
MO=0

1/2W(1/2D)=P(1/2D)+1/2W(1/2d)
Problem 339
The differential chain hoist shown in Fig. P-339
consists of two concentric pulleys rigidly fastened
together. The pulleys form two sprockets for an
endless chain looped over them in two loops. In one
loop is mounted a movable pulley supporting a load
W. Neglecting friction, determine the maximum
load W that can just be raised by a pull P supplied
as shown.
1/4WD=1/2PD+1/4Wd

1/4WD1/4Wd=1/2PD

1/4(Dd)W=1/2PD

W=[1/2PD]/{1/4(Dd){

W=[2PD]/{Dd} answer

Problem 342
The wheel loads on a jeep are given in Fig. P-342.
Determine the distance x so that the reaction of the
beam at A is twice as great as the reaction at B.

Solution 340

Problem 340
For the system of pulleys shown in Fig. P-340,
determine the ratio of W to P to maintain
equilibrium. Neglect axle friction and the weights of
the pulleys.
From the lowermost pulley

answer

Problem 341
If each pulley shown in Fig. P-340 weighs 36 kg
and W = 720 kg, find P to maintain equilibrium.

Solution 341

From pulley A

From pulley B

From pulley C

Thus, answer

Solution 342
The reaction at A is twice as the reaction at B are on the same plane.
RA=2RB

FV=0
1. Find the reaction at A.
RA+RB=600+200 A. 900 N
B. 800 N
2RB+RB=800 C. 1400 N
D. 2400 N
3RB=800
2. Find the reaction at B.
RB=266.67 lb A. 900 N
B. 800 N
C. 1400 N
MA=0
D. 2400 N
600x+200(x+4)=15RB
3. Find the reaction at C.
600x+200x+800=15(266.67) A. 900 N
B. 800 N
800x=3200 C. 1400 N
D. 2400 N
x=4 ft answer
Solution

Plank DEB
MD=0
Reactions of Tripod Made from
Wood Planks 4RB=2(1004)+2RE

Situation 2RB=400+RE eq. (1)


In the figure shown, each plank carries a uniform
load of 100 N/m throughout its length. The supports
4RF=2(1004)+2RD

2RF=400+RD eq. (5)

MB=0

4RD=2(1004)+4(3500)+2RE

2RD=7400+RE eq. (2)


MF=0

4RA=2(1004)+2RD
Plank EFC
2RA=400+RD eq. (6)
MC=0

4RE=2(1004)+2RF
eq. (1) - eq. (2)
2RE=400+RF eq. (3)
2RB2RD=7000

RD=RB+3500

From eq. (6)


2RA=400+(RB+3500)

2RARB=3900 eq. (7)

ME=0 eq. (3) - eq. (4)

4RC=2(1004)+2RF 2RE2RC=0

2RC=400+RF eq. (4) RE=RC

From eq. (1)


Plank FDA
2RB=400+RC
MA=0
2RBRC=400 eq. (8)
eq. (5) - eq. (6)

2RF2RA=0

RF=RA

From eq. (4)


2RC=400+RA

RA+2RC=400 eq. (9)

Using the Calculator for eq. (7), eq. (8), and eq. (9) MBG=0

RA=2400 N Answer: [ D ] bRA+(4+b)RC=2bP+2bW

RB=900 N Answer: [ A ] 1RA+(4+1)RC=2(1)(3500)+2(1)(1200)

RC=1400 N Answer: [ C ] RA+5RC=9400 eq. (1)

MBI=0
Another Solution
3aRA+aRC=2aP+(a+1/3a)W
P=3500 N
3RA+RC=2P+4/3W
W=3[100(4)]=1200 N
3RA+RC=2(3500)+4/3(1200)

a=2sin60=3 m 3RA+RC=8600 eq. (2)

b=2cos60=1 m
From eq. (1) and eq. (2)
RA=2400 N

RC=1400 N

MHI=0

4RA+(4+b)RB=3P+3W

4RA+(4+1)RB=3(3500)+3(1200)
4RA+5RB=14,100 sin=(1/7) 21

4(2400)+5RB=14,100

RB=900 N a=xsin60=27sin60=21 m

b=4sin=4((1/7) 21)=(4/7) 21m


Summary
c=1/3a=(1/3) 21 m
RA=2400 N Answer: [ D ]
d=2sin=2([1/14] 21)=[1/7] 21
RB=900 N Answer: [ A ]
e=4sin=4([1/14] 21)=[2/7] 21
RC=1400 N Answer: [ C ]

Another Solution

P=3500 N

W=3[100(4)]=1200 N

From triangle BEC in the figure below

x2=22+422(2)(4)cos120

x2=28

x=27 m

MBC=0
x/sin120=2/sin
aRA=bP+cW
sin=2sin120/x
21RA=[4/7] 21 (3500)+[1/3] 21(1200)
sin=2sin120/[27]
RA=[4/7](3500)+(1200)
sin=(1/14) 21
RA=2400 N

x/sin120=4/sin
MAC=0
sin=[4sin120]/x
aRB=dP+cW
sin=[4sin120]/27
21RB=[1/7] 21(3500)+[1/3] 21(1200)
RB=[1/7](3500)+[1/3](1200) D. 2,010 N

RB=900 N 2. Find the reaction at B.


A. 16,848 N
B. 6,716 N
MAB=0 C. 13,372 N
D. 2,010 N
aRC=eP+cW
3. Find the reaction at C.
21RC=[2/7] 21(3500)+[1/3] 21(1200)
A. 16,848 N
B. 6,716 N
RC=[2/7](3500)+[1/3](1200)
C. 13,372 N
RC=1400 N D. 2,010 N

Summary
Solution
RA=2400 N Answer: [ D ]
W1=3600[{1/2}(2.20)(5.40)]=21,384 N
RB=900 N Answer: [ A ]
W2=3600[{1/2}(1.60)(5.40)]=15,552 N
RC=1400 N Answer: [ C ]
W=W1+W2=36,936 N

Support Reactions of a Trapezoidal


Slab with Three Points of Support
Situation
A reinforced concrete slab in the shape of an
isosceles trapezoid weighs 3600 N/m2. It is
supported on the three points as shown in the figure.

MR1=0

3.6RC=(5.41.81.2)W1+(1.81.2)W2

3.6RC=2.4(21,384)+0.6(15,552)

RC=16,848 N

1. Find the reaction at A.


A. 16,848 N
B. 6,716 N
C. 13,372 N
Summary:
RA = 6,715.64 N Answer: [ B ]
RB = 13,372.36 N Answer: [ C ]
RC = 16,848 N Answer: [ A ]

Equilibrium of Non-Concurrent
Force System

MC=0

3.6R1=(1.80.6)W1+(5.41.80.6)W2 Problem 346


A boom AB is supported in a horizontal position by
3.6R1=1.2(21,384)+3(15,552) a hinge A and a cable which runs from C over a
small pulley at D as shown in Fig. P-346. Compute
R1=20,088 N the tension T in the cable and the horizontal and
vertical components of the reaction at A. Neglect
RA+RB=20,088 N Equation (1) the size of the pulley at D.

MR2=0

1.1RA=0.2(36,936)

RA=6,715.64 N

From Equation (1)


6,715.64+RB=20,088
Solution 346
RB=13,372.36 N
MA=0

4(2/5T)=2(200)+6(100)
T=279.51 lb answer

FV=0

AV+[2/5]T=200+100

AV+[2/5](279.51)=300
Solution 347
AV=50 lb answer

FH=0

AH=1/5T

AH=1/5 (279.51)

AH=125 lb answer

Problem 347
Repeat Problem 346 if the cable pulls the boom AB
into a position at which it is inclined at 30 above
the horizontal. The loads remain vertical.
sin60=x4

x=4sin60

tan=6x

tan=64sin60
tan=3

=60

Because = 60, T is perpendicular to AB.


MA=0

4T=200(2cos30)+100(6cos30)

T=216.51 lb answer

FH=0

AH=Tcos

AH=216.51cos60
Solution 348
AH=108.25 lb answer
Length of DF
LDF2=42+32
FV=0
LDF2=25
AV+Tsin=200+100
LDF=5 m
AV+216.51sin60=200+100

AV=112.50 lb answer
Weights of members
WAB=6(5)=30 kN

WCE=6(5)=30 kN
Problem 348
The frame shown in Fig. P-348 is supported in
WDF=5(5)=25 kN
pivots at A and B. Each member weighs 5 kN/m.
Compute the horizontal reaction at A and the
horizontal and vertical components of the reaction
at B.
the reactions.

Solution 349
MB=0
MB=0
6AH=3WCE+2WDF+6(200)
24AV+16(240)=36(400)+12(600)
6AH=3(30)+2(25)+6(200)
AV=740 lb answer
AH=223.33 kN answer

FH=0

BH=AH

BH=223.33 kN answer

FV=0

BV=WAB+WCE+WDF+200

BV=30+30+25+200

BV=285 kN answer

Problem 349 MA=0


The truss shown in Fig. P-349 is supported on roller
24BV+12(400)=16(240)+12(600)
at A and hinge at B. Solve for the components of
BV=260 lb answer 7BV+3(60)=3(120)+4(30)+11(50)

BV=121.43 kN
FH=0

BH=240 lb answer FH=0

Problem 350 BH=30 kN


Compute the total reactions at A and B for the truss
shown in Fig. P-350.
RB=BH2+BV2=302+121.432

RB=125.08 kN

tanBx=BV/BH=121.43/30

Bx=76.12

Thus, RB=12.08 kN up to the left


at 76.12 from horizontal. answer

Problem 351
Solution 350 The beam shown in Fig. P-351 is supported by a
hinge at A and a roller on a 1 to 2 slope at B.
MB=0
Determine the resultant reactions at A and B.
7RA+4(30)+4(50)=10(60)+4(120)

RA=108.57 kN answer

MA=0
Solution 351 Another Solution

From Equilibrium of Concurrent Force System,


three coplanar forces in equilibrium are concurrent.

MA=0

4(2/5RB)=3(40)
y/1=2/1
RB=33.54 kN
y=2 m

MB=0
tanAx=y/3
4AV=1(40)
tanAx=2/3
AV=10 kN
Ax=33.69 okay

FH=0
tanBx=2/1
AH=1/5RB=1/5 (33.54)
Bx=63.43
AH=15 kN

=90Ax=56.31
2 2 2 2
RA=AH +AV =15 +10
=90Bx=26.57
RA=18.03 kN
=Ax+Bx=97.12

tanAx=AV/AH=10/15
RA/sin=RB/sin=40/sin
Ax=33.69
RA/sin26.57=RB/sin56.31=40/sin97.12

Thus, RA=18.03 kN up to the right RA=18.03 kN okay


at 33.69 from horizontal. answer
RB=33.54 kN okay
Problem 352 BH=200cos30
A pulley 4 ft in diameter and supporting a load 200
lb is mounted at B on a horizontal beam as shown in BH=173.20 lb
Fig. P-352. The beam is supported by a hinge at A
and rollers at C. Neglecting the weight of the beam,
determine the reactions at A and C. From FBD of beam
MA=0

8RC=4BV

8RC=4(100)

RC=50 lb answer

Solution 352 MC=0

From FBD 8AV=4BV


of pulley
T=200 lb 8AV=4(100)

AV=50 lb
FV=0

FH=0

AH=BH

BV+Tsin30=200 AH=173.20 lb

BV+200sin30=200
RA=AH2+AV2
BV=100 lb
RA=173.202+502

FH=0 RA=180.27 lb

BH=Tcos30
tanAx=AV/AH Rx=480.38 kN answer

tanAx=50/173.20

Ax=16.1

Thus, RA=180.27 lb up to the right


at 16.1 from horizontal. answer

Problem 353
The forces acting on a 1-m length of a dam are
shown in Fig. P-353. The upward ground reaction
varies uniformly from an intensity of p1 kN/m to p2
kN/m at B. Determine p1 and p2 and also the
horizontal resistance to sliding.

FV=0

Ry=W+Fsin30

Ry=2400+600sin30

Ry=2700 kN

Righting moment
MR=11(2400)+4(600)

MR=28800 kNm

Overturning moment
MO=6(1000)
Solution 353
MO=6000 kNm
Horizontal resistance to sliding
FH=0
MB=0
Rx+Fcos30=1000
xRy=MRMO
Rx+600cos30=1000
x(2700)=288006000
x=8.44 m to the left of B

Eccentricity
e=1/2Bx=1/2(18)8.44

e=0.56 m

Solution 354

MA=0

24RB+3(20)+3(1/5)(22.4)=18(2/5)(22.4)+18(30)
+12(20)+6(10)

RB=46.27 kN answer

Foundation pressure (See Analysis of Gravity Dam


for more information)
p=Ry/B(16e/B)

p1=2700/18[1[6(0.59)]/18]=122 kN/m answ


er

p2=2700/18[1+[6(0.59)]/18]=178 kN/m answ


er
MB=0

24AV=3(20)+3(1/5)(22.4)+6(2/5)(22.4)+6(30)+1
Problem 354 2(20)+18(10)
Compute the total reactions at A and B on the truss
shown in Fig. P-354. AV=33.76 kN

FH=0

AH=20+[1/5](22.4)

AH=30.02 kN

RA=AH2+AV2
RA=30.022+33.762 cos=4.5/AC

RA=45.18 kN AC=4.5/cos=4.5/cos26.56

AC=5.03 m
tanAx=AV/AH

tanAx=33.76/30.02

tanAx=48.36

Thus, RA=45.18 kN up to the right


at 48.36 from horizontal. answer

cos=AC/AD

Problem 355 AD=AC/cos=5.03/cos26.56


Determine the reactions at A and B on the Fink
truss shown in Fig. P-355. Members CD and FG are AD=5.626 m
respectively perpendicular to AE and BE at their
midpoints.
FB=AD

FB=5.626 m

MB=0

18(RAcos30)=12(184.5)+(40cos)(FB)+40(FB)+
20(18AD)

18(RAcos30)=12(13.5)+(40cos26.56)(5.626)+40(
5.626)+20(185.626)

15.59RA=835.81

RA=53.61 kN answer
Solution 355

tan=4.5/9 FH=0

=26.56 BH+40sin=RAsin30

BH+40sin26.56=53.61sin30
BH=8.92 kN answer Notice also that triangle ABD is an equilateral
triangle of sides 6 m.

MA=0

18BV=12(4.5)+(40cos)(18FB)+40(18FB)+20(A
D)

18BV=12(4.5)+(40cos26.56)(185.626)+40(185.
626)+20(5.626)

18BV=1104.20

BV=61.34 kN answer

Problem 356
The cantilever truss shown in Fig. P-356 is
supported by a hinge at A and a strut BC.
Determine the reactions at A and B.
MA=0

6(RBcos30)=3(10)+6(10)+9(10)

RB=203 kN

RB=34.64 kN answer

FH=0

AH+4(10sin30)=RBcos30

AH+4(10sin30)=203cos30

AH=10 kN

Solution 356
FV=0
From right triangles ACD and ACB.
cos30=AC/6=AC/AB AV+RBsin30=4(10cos30)

AB=6 m AV+203sin30=4(10cos30)

AV=103 kN
AV=17.32 kN Distance AB
AB=82+82=82 m

RA=AH2+AV2

RA=102+(103)2

RA=20 kN

tanAx=AV/AH

tanAx=[103]/10

Ax=60

Thus, RA=20 kN up to the right at 60 from


horizontal. answer

FH=0
Problem 357
T=RBcos45
The uniform rod in Fig. P-357 weighs 420 lb and
has its center of gravity at G. Determine the tension
in the cable and the reactions at the smooth surfaces
MA=0
at A and B.
2T+6(420)=82RB

2T+2520=82RB

T+1260=42RB

RBcos45+1260=42RB

4.9497RB=1260

RB=254.56 lb answer

T=254.56cos45

T=180 lb answer

Solution 357 FV=0


RA+RBsin45=420

RA+254.56sin45=420

RA=240 lb answer

Problem 358
A bar AE is in equilibrium under the action of the
five forces shown in Fig. P-358. Determine P, R, and
T.

MA=0

10T+4(4)R=4(60)+3(3)(40)

10T+16R=600

5T+8R=300

5T+8(604/5T)=300

7/5T+480=300

7/5T=180

T=128.57 kN up to the left answer

Solution 358 R=604/5(128.57)

FV=0 R=42.86 kN

4/5T+R=60 R=42.86 kN downward answer

R=604/5T
FH=0

P+3/5T=40

P+3/5(128.57)=40

P=37.14 kN
P=37.14 kN to the right answer From the Force Polygon

Problem 359
A 4-m bar of negligible weight rests in a horizontal
position on the smooth planes shown in Fig. P-359.
Compute the distance x at which load T = 10 kN
should be placed from point B to keep the bar RA/sin45=[20+10]/sin105
horizontal.
RA=21.96 kN

From the Free Body Diagram


MB=0

4(RAcos30)=20(3)+10x

4(21.96cos30)=20(3)+10x

10x=16.072

x=1.61 m answer
Solution 359

Problem 360
Referring to Problem 359, what value of T acting at
x = 1 m from B will keep the bar horizontal.
Solution 360 Problem 361
Referring to Problem 359, if T = 30 kN and x = 1
m, determine the angle at which the bar will be
inclined to the horizontal when it is in a position of
equilibrium.

From the Force Polygon

Solution 361

RA/sin45=[20+T]/sin105

RA=0.732(20+T)

RA=14.641+0.732T

From the Free Body Diagram


MB=0 From the Force Polygon
RA/sin45=[20+30]/sin105
4(RAcos30)=3(20)+1(T)
RA=36.60 kN
4(14.641+0.732T)cos30=60+T

50.7179+2.5357T=60+T

1.5357T=9.2821

T=6.04 kN answer
Solution 506

Hide Click here to show or hide the solution


Part (a) - Force is applied horizontally

FV=0

N=400 lb
From the Free Body Diagram
MB=0
f=N=0.40(400)
(4cos)(RAcos30)=(4sin)(RAsin30)+(3cos)(20)
+(1cos)(30)
f=160 lb
(4cos)(36.60cos30)=(4sin)(36.60sin30)+(3cos
)(20)+(1cos)(30)
FH=0
126.7861cos=73.2sin+60cos+30cos
P=f
36.7861cos=73.2sin
P=160 lb answer
36.7861/73.2=sin/cos
Part (b) - Downward force at 30 from the
tan=0.5025423497
horizontal
=26.68 answer

Friction
Problem 506 FV=0
A 400 lb block is resting on a rough horizontal
surface for which the coefficient of friction is 0.40. N=400+Psin30
Determine the force P required to cause motion to
impend if applied to the block (a) horizontally or (b) N=400+0.5P
downward at 30 with the horizontal. (c) What
minimum force is required to start motion?
f=N=0.40(400+0.5P)
f=160+0.2P Part (c) - Minimum force required to cause
impending motion
FV=0
FH=0

Pcos30=f

Pcos30=160+0.2P

0.666P=160
N=400Psin
P=240.23 lb answer

f=N=0.40(400Psin)
Another Solution for Part (b)
tan=
f=1600.40Psin
tan=0.40
FH=0
=21.80
Pcos=f

Pcos=1600.40Psin

Pcos+0.40Psin=160

(cos+0.40sin)P=160

P=160/[cos+0.40sin]
=90(30+)

=90(30+21.80) To minimize P, differentiate then equate to zero


dP/d=[160(sin+0.40cos)]/(cos+0.40sin)2=
=38.20 0

sin0.40cos=0
P/sin=400/sin
sin=0.40cos
P/sin21.80=400/sin38.20
tan=0.40
P=240.21 lb okay!
=21.80

Minimum value of P
Pmin=160/[cos21.80+0.40sin21.80]
Pmin=148.56 lb answer Fy=0

Problem 507
The 2225-N block shown in Fig. P-507 is in contact
with 45 incline. The coefficient of static friction is
0.25. Compute the value of the horizontal force P
N=2225cos45+Psin45
necessary to (a) just start the block up the incline or
(b) just prevent motion down the incline. (c) If P =
N=1573.31+0.7071P
1780 N, what is the amount and direction of the
friction force?
f=N=0.25(1573.31+0.7071P)

f=393.33+0.1768P

Fx=0

Pcos45=f+2225sin45

Pcos45=(393.33+0.1768P)+2225sin45

0.5303P=1966.64

P=3708.55 N answer
Solution 507

Part (b) Force P to just prevent the block to


Part (a) Force P to just start the block to move
slide down the incline
up the incline
In this case, the force P is not pushing the block
The force P is pushing the block up the incline. The
upward, it simply supports the block not to slide
push is hard enough to overcome the maximum
downward. Therefore, the total force that prevents
allowable friction causing an impending upward
the block from sliding down the plane is the sum of
motion.
the component of P parallel to the incline and the
upward friction force.
Fy=0

N=2225cos45+Psin45
Wx=2225sin45=1573.31 N
N=1573.31+0.7071P
Px=1780cos45=1258.65 N

f=N=0.25(1573.31+0.7071P)
Wx > Px, thus, f is upward.
f=393.33+0.1768P Fx=0

f+Px=Wx
Fx=0
f+1258.65=1573.31
Pcos45+f=2225sin45
f=314.66 N upward answer
Pcos45+(393.33+0.1768P)=2225sin45

0.8839P=1179.98
Problem 508
P=1335 N answer The 200-lb block shown in Fig. P-508 has
impending motion up the plane caused by the
horizontal force of 400 lb. Determine the coefficient
Part (c) Force P = 1780 N of static friction between the contact surfaces.
If Px = Wx, there will be no friction under the block.
If Px > Wx, friction is going downward to help Wx
balance the Px. If Px < Wx, friction is going upward
to help Px balance the Wx. In this problem, the
maximum available friction is not utilized by the
system.

Solution 508
Fy=0

N=400sin30+200cos30

N=373.20 lb
Solution 509

Fx=0 Neglecting friction

f+200sin30=400cos30

f=246.41 lb

f=N

246.41=(373.20)

=0.66 answer

Problem 509
The blocks shown in Fig. P-509 are connected by
flexible, inextensible cords passing over frictionless TA=3000sin=3000(3/5)
pulleys. At A the coefficients of friction are s =
0.30 and k = 0.20 while at B they are s = 0.40 and TA=1800 N
k = 0.30. Compute the magnitude and direction of
the friction force acting on each block.
TB=2000sin=2000(4/5)

TB=1600 N

2TA is greater than TB, thus, the system will move


to the left if contact surfaces are frictionless.
Considering friction Blocks are moving to the left (Use k)
The angle of static friction at A, A = arctan 0.30 = fA=0.20NA=0.20(2400)=480 N answer
16.70, is not enough to hold the block from sliding
the incline of angle = arctan (3/4) = 36.87 from fB=0.30NB=0.30(1200)=360 N answer
horizontal. If TB is insufficient to hold 2TA
statically the system will move to the left,
otherwise, the system is stationary.
Problem 510
What weight W is necessary to start the system of
blocks shown in Fig. P-510 moving to the right?
The coefficient of friction is 0.10 and the pulleys
are assumed to be frictionless.

Assume the blocks are stationary (use s) Solution 510


NA=3000cos=3000(4/5)=2400 N
N1=60 kN
fA=0.30NA=0.30(2400)=720 N
f1=N1=0.10(60)=6 kN
TA=3000sinfA=3000(3/5)720=1080 N
T1=f1=6 kN
2TA=2160 N

NB=2000cos=2000(3/5)=1200 N

fB=0.40NB=0.40(1200)=480 N

TB=2000sin+fB=2000(4/5)+480=2080 N

TB < 2TA. TB is insufficient to hold the system in


static equilibrium, thus, the blocks are moving to
the left.
N2=40cos30=34.64 kN

f2=N2=0.10(34.64)=3.46 kN

W=40sin30+T1+f2

W=20+6+3.46

W=29.46 kN answer

Sum up forces parallel to the incline


T=f2+10sin30

Problem 511
T=1.732+10sin30
Find the least value of P required to cause the
system of blocks shown in Fig. P-511 to have T=6.732 kN
impending motion to the left. The coefficient of
friction under each block is 0.20.
From the FBD of 30 kN block on the horizontal
plane
Sum up vertical forces
N1+Psina=30

N1=30Psin

Amount of friction
f1=N1=0.20(30Psin)

f1=60.20Psin
Solution 511

From the FBD of 10 kN block on the inclined Sum up horizontal forces


plane Pcos=f1+T
Sum up forces normal to the incline
N2=10cos30=8.66 kN Pcos=(60.20Psin)+6.732

Amount of friction
Pcos+0.20Psin=12.732
f2=N2=0.20(8.66)=1.732 kN P(cos+0.20sin)=12.732

P=12.732/[cos+0.20sin]
To minimize P, differentiate then equate to zero Sliding up the incline
dP/d={12.732(sin+0.20cos)}/(c Fy=0
os+0.20sin)2=0
N=Wcos=4/5W
sin+0.20cos=0 f=N=0.30(4/5W)=6/25W

sin=0.20cos

tan=0.20

=11.31

Thus,
Pmin=12.732/[cos11.31+0.20sin11.31]

Pmin=12.5 kN answer

Problem 512
A homogeneous block of weight W rests upon the
incline shown in Fig. P-512. If the coefficient of Fx=0
friction is 0.30, determine the greatest height h at
which a force P parallel to the incline may be P=Wsin+f
applied so that the block will slide up the incline
without tipping over. P=3/5W+6/25W

P=21/25W

Tipping over

Solution 512
MA=0

Ph=40(Wsin)+20(Wcos)

21/25Wh=40(3/5W)+20(4/5W)

h=47.62 cm answer

Problem 513
In Fig. P-512, the homogeneous block weighs 300
kg and the coefficient of friction is 0.45. If h = 50
cm, determine the force P to cause motion to f=N=0.45(240)
impend.
f=108 kg

Fx=0

P+f=300sin

P+108=300(3/5)

P=72 kg answer

Problem 514
The 10-kN cylinder shown in Fig. P-514 is held at
rest on the 30 incline by a weight P suspended
from a cord wrapped around the cylinder. If
slipping impends, determine P and the coefficient of
Solution 513 friction.

Fy=0

N=300cos=300(4/5)

N=240 kg
f=P

f=10 kN

Fy=0

N=10cos30+Pcos30

N=10cos30+10cos30

N=17.32 kN

f=N
Solution 514
10=(17.32)

=0.577 answer

Problem 515
Block A in Fig. P-515 weighs 120 lb, block B
weighs 200 lb, and the cord is parallel to the
incline. If the coefficient of friction for all surfaces
in contact is 0.25, determine the angle of the
incline of which motion of B impends.

a=10cos60=5 cm

b=10a=5 cm

MA=0

Pb=Wcyla

P(5)=10(5)

P=10 kN answer

MO=0

10f=10P
Solution 515 Problem 516
Referring to Fig. P-515 if the coefficient of friction
Hide Click here to show or hide the solution is 0.60 and = 30, what force P applied to B acting
down and parallel to the incline will start motion?
N1=120cos
What is the tension in the cord attached to A?

f1=0.25(120cos)=30cos

Solution 516
N2=N1+200cos

N2=120cos+200cos

N2=320cos

f2=0.25(320cos)=80cos

f1+f2=200sin

30cos+80cos=200sin

110cos=200sin

100/200=sin/cos
N1=120cos30=103.92 lb
tan=1120
f1=0.60(103.92)=62.35 lb
=28.81 answer
N2=N1+200cos30 Solution 519

N2=103.92+173.20

N2=277.12 lb

f2=0.60(277.12)=166.27 lb

P+200sin30=f1+f2

P+100=62.35+166.27

P=128.62 lb answer
Sum up forces normal to the incline in block B
NB=2700cos60+Csin30
T=f1+120sin30
NB=1350+0.5C
T=62.35+60

T=122.35 lb answer
Amount of friction for impending motion of block
B
fB=NB=0.25(1350+0.5C)
Problem 519
fB=337.5+0.125C
In Fig. P-519, two blocks are connected by a solid
strut attached to each block with frictionless pins. If
the coefficient of friction under each block is 0.25
Sum up all forces parallel to the incline in block B
and B weighs 2700 N, find the minimum weight of
fB+Ccos30=2700sin60
A to prevent motion.
(337.5+0.125C)+Ccos30=2700sin60

0.991C=2000.77

C=2018.89 N

Summation of vertical forces acting on block A


NA=WA+Csin30

NA=WA+2018.89sin30

NA=WA+1009.44
Amount of friction under block A at impending Apply Sine law to force polygon B
motion C/sin(60)=2700/sin(60+)
fA=NA=0.25(WA+1009.44)
C/sin45.96=2700/sin74.04
fA=0.25WA+252.36
C=2018.72 N
Summation of horizontal forces on block A
fA=Ccos30 Sine law to force triangle A
WA/sin(60)=C/sin
0.25WA+252.36=2018.89cos30
WA/sin45.96=2018.72/sin14.04
WA=5984.20 N answer
WA=5981.75 N answer

Another Solution for Problem 519


Problem 520
tan= Referring to Fig. P-519, block A weighs 4 kN and B
weighs 3 kN. If = 0.20 under B, compute the
tan=0.25 minimum coefficient of friction under A to prevent
motion.
=14.04

Solution 520

60=45.96

60+=74.04
fA=23.79cos30

fA=20.60 kN

Coefficient of friction at A
fA=ANA

20.60=A(51.895)

A=0.397 answer

Summation of forces perpendicular to the inclined


Another Solution to Problem 520
plane in block B
NB=30cos60+Csin30
tanB=B
NB=15+0.5C
tanB=0.20

Amount of friction in block B at impending motion B=11.31


fB=BNB=0.20(15+0.5C)

fB=3+0.10C

fB+Ccos30=3sin60

(3+0.10C)+Ccos30=30sin60

0.966C=22.98

C=23.79 kN

Summation of vertical forces in block A


NA=40+Csin30

NA=40+23.79sin30

NA=51.895 kN

Summation of horizontal forces in block A


fA=Ccos30
60B=48.69

60+B=71.31

Sine law to the force polygon B


C/sin(60B)=30/sin(60+B)

C/sin48.69=30/sin71.31

C=23.79 kN

Sine law to force polygon A


Solution 521
40/sin(60A)=C/sinA
Sum up vertical forces in block A
40/sin(60A)=23.79/sinA NA=400+Csin30

40sinA=23.79sin(60A)
Friction force at block A
40sinA=23.79(sin60cosAcos60sin fA=NA=0.30(400+Csin30)
A)
fA=120+0.15C
40sinA=20.60cosA11.895sinA

51.895sinA=20.60cosA

tanA=0.3970

A=0.397 answer

Problem 521
In Fig. P-519, if = 0.30 under both blocks and A
weighs 400 lb, find the maximum weight of B that
can be started up the incline by applying to A a
rightward force P of 500 lb.

Sum up horizontal forces in block A


fA+Ccos30=500

(120+0.15C)+Ccos30=500
1.016C=380

C=374 lb

Sum up forces normal to the incline in block B


NB=Csin30+WBcos60

NB=374sin30+WBcos60

NB=187+0.5WB

Amount of friction force under block B Solution 522


fB=NB=0.30(187+0.5WB)
Summation of forces normal to the incline in the
fB=56.1+0.15WB 200-lb block
N2=350cos45+Ccos75

Sum up forces parallel to the inclined plane in block N2=247.49+0.2588C


B
fB+WBsin60=Ccos30
Amount of friction under the 200-lb block
(56.1+0.15WB)+WBsin60=374cos30 f2=N2=0.20(247.49+0.2588C)

1.016WB=267.79 f2=49.498+0.0518C

WB=263.57 lb answer
Summation of forces parallel to the incline in the
200-lb block
f2+350sin45=Csin75
Problem 522
The blocks shown in Fig. P-522 are separated by a (49.498+0.0518C)+350sin45=Csin75
solid strut which is attached to the blocks with
frictionless pins. If the coefficient of friction for all 0.9141C=296.985
surfaces is 0.20, determine the value of horizontal
C=324.89 lb
force P to cause motion to impend to the right.
Assume that the strut is a uniform rod weighing 300
lb.
Summation of all vertical forces acting in the 400-lb
block
N1=550+Csin30

N1=550+324.89sin30

N1=712.45 lb
Amount of friction under the 400-lb block
f1=N1=0.20(712.45)

f1=142.50 lb

Solving for the required P by summing up


horizontal forces in the 400-lb block
P=f1+Ccos30

P=142.50+324.89cos30

P=423.85 lb answer

Problem 523
A force of 400 lb is applied to the pulley shown in
Fig. P-523. The pulley is prevented from rotating by
a force P applied to the end of the brake lever. If the
coefficient of friction at the brake surface is 0.20,
determine the value of P.

MO=0

20f=10(400)

f=200 lb

f=N

200=0.20N

N=1000 lb

MA=0
Solution 523
48P+8f=16N

48P+8(200)=16(1000)

48P=14400

P=300 lb answer
Problem 524
A horizontal arm having a bushing of 20 mm long is
slipped over a 20-mm diameter vertical rod, as
shown in Fig. P-524. The coefficient of friction
between the bushing and the rod is 0.20. Compute
the minimum length L at which a weight W can be
placed to prevent the arm from slipping down the
rod. Neglect the weight of the arm.

Amount of friction force


f=N=0.20(0.10WL)

f=0.02WL

FV=0

f=W

0.02WL=W
Solution 524
L=50 mm answer
MO=0

10N=WL
Problem 525
N=0.10WL A uniform ladder 4.8 m ft long and weighing W lb
is placed with one end on the ground and the other
against a vertical wall. The angle of friction at all
Note: contact surfaces is 20. Find the minimum value of
the sum of f will pass through point O, thus, f has the angle at which the ladder can be inclined with
no moment effect at O. the horizontal before slipping occurs.

Solution 525
Coefficient of MB=0
friction
W(2.4cos)+fA(4.8sin)=NA(4.8cos)

Wcos+2fAsin=2NAcos

W+2fAtan=2NA

W+2(0.3214W)tan=2(0.883W)

1+0.6428tan=1.766

0.6428tan=0.766

tan=1.191 661 481


=tan=tan20

=0.364 =50 answer

Friction forces at each end of the ladder


fA=NA=0.364NA Another Solution 525

fB=NB=0.364NB sin20=RB/W

RB=Wsin20
FH=0

NB=fA
RBH=RBcos20=Wsin20cos20
NB=0.364NA
RBV=RBsin20=Wsin20sin20=Wsin220

FV=0

NA+fB=W

NA+0.364NB=W

NA+0.364(0.364NA)=W

1.1325NA=W

NA=0.883W

Thus,
fA=0.364(0.883W)

fA=0.3214W MA=0
RBH(4.8sin)+RBV(4.8cos)=W(2.4cos)

4.8RBHsin=2.4Wcos4.8RBVcos

2RBHsin=Wcos2RBVcos

2RBHsin=(W2RBV)cos

Sin/cos=[W2RBV]/2RBH

tan=[W2RBV]/2RBH

tan=[W2Wsin220]/2Wsin20cos20

tan=[12sin220]/2sin20cos20
Coefficient of friction
=tan
=arctan([12sin220]/2sin20cos20)
=tan15
=arctan1.191753593

=50 answer
Amount of friction at contact surfaces
fA=NA=NAtan15

Problem 526 fB=NB=NBtan15


A ladder 6 m long has a mass of 18 kg and its center
of gravity is 2.4 m from the bottom. The ladder is
FV=0
placed against a vertical wall so that it makes an
angle of 60 with the ground. How far up the ladder
NA+fB=18+72
can a 72-kg man climb before the ladder is on the
verge of slipping? The angle of friction at all
NA=90fB
contact surfaces is 15.
NA=90NBtan15

Solution 526
FH=0

fA=NB

NAtan15=NB

(90NBtan15)tan15=NB

90tan15NBtan215=NB

90tan15=NB+NBtan215
NB(1+tan215)=90tan15 Solution 527

NB=90tan15/[1+tan215] FH=0

NB=22.5 kg

fB=22.5tan15

fB=6.03 kg

MA=0

NB(6sin60)+fB(6cos60)=18(2.4cos60)+72(xcos6
0) NAcos15=NBcos75

NB(6tan60)+6fB=18(2.4)+72x NA=0.2679NB

6(22.5)tan60+6(6.03)=43.2+72x
FV=0
72x=226.81
NAsin15+NBsin75=30
x=3.15 m answer
(0.2679NB)sin15+NBsin75=30
Problem 527
A homogeneous cylinder 3 m in diameter and 1.0353NB=30
weighing 30 kN is resting on two inclined planes as
shown in Fig. P-527. If the angle of friction is 15 NB=28.98 kN
for all contact surfaces, compute the magnitude of
the couple required to start the cylinder rotating
counterclockwise. NA=0.2679(28.98)

NA=7.76 kN

=tan15=0.2679

fA=NA=0.2679(7.76)

fA=2.08 kN

fB=NB=0.2679(28.98)
fB=7.76 kN

Problem 529
Required couple As shown in Fig. P-529, a homogeneous cylinder 2
C=Mcenter m in diameter and weighing 12 kN is acted upon by
a vertical force P. Determine the magnitude of P
C=1.5(fA+fB)=1.5(2.08+7.76) necessary to start the cylinder turning. Assume that
= 0.30.
C=14.76 kNm answer

Problem 528
Instead of a couple, determine the minimum
horizontal force P applied tangentially to the left at
the top of the cylinder described in Prob. 527 to
start the cylinder rotating counterclockwise.

Solution 528

Solution 529

When the cylinder starts to turn due to P, the normal


force under horizontal surface is zero. See the free
body diagram below.

1.5F=1.5fA+1.5fB

F=fA+fB

F=2.08+7.76

F=9.84 kN answer
x=(1)(sin60)
x=(1/2) 3 m

MA=0

(1+x)P=12x

(1+[1/2]3)P=12([1/2] 3)

1.866P=10.392

P=5.569 kN answer

Problem 530
A plank 10 ft long is placed in a horizontal position MA=0
with its ends resting on two inclined planes, as
Px=(RBcos50)(10)
shown in Fig. P-530. The angle of friction is 20.
Determine how close the load P can be placed to
Px=(0.4375Pcos50)(10)
each end before slipping impends.
x=2.81 ft answer

The plank impends to the left


RA/sin10=P/sin105

RA=0.1798P

Solution 530

The plank impends to the right


RB/sin25=P/sin105

RB=0.4375P

MB=0
Py=(RAcos65)(10) 2WLcos30cos=WLcos+2WLsin30sin

Py=(0.1798Pcos65)(10) 2cos30cos=cos+2sin30sin

y=0.76 ft answer 2cos30coscos=2sin30sin

(2cos301)cos=2sin30sin

Problem 531 [2cos301]/2sin30=sin/cos


A uniform plank of weight W and total length 2L is
placed as shown in Fig. P-531 with its ends in tan=31
contact with the inclined planes. The angle of
friction is 15. Determine the maximum value of the =arctan(31)
angle at which slipping impends.
=36.21 answer

Problem 532
In Fig. P-532, two blocks each weighing 1.5 kN are
connected by a uniform horizontal bar which
weighs 1.0 kN. If the angle of friction is 15 under
each block, find P directed parallel to the 45
incline that will cause impending motion to the left.

Solution 531

The force polygon below is isosceles, thus, RB=W

Solution 532

=tan
MA=0
=tan15
(RBcos30)(2Lcos)=W(Lcos)+(RBsin30)(2Lsin
)

(Wcos30)(2Lcos)=W(Lcos)+(Wsin30)(2Lsin)
NB=2.8284 kN

Amount of friction under block B


fB=NB=tan15(2.8284)

fB=0.7578 kN

Summation of forces on block B parallel to the 45


incline
P+2sin45=fB+Csin45

Summation of forces on block A normal to the 30 P+2sin45=0.7578+2sin45


incline
P=0.7578 kN answer
NA=2cos30+Ccos60

NA=2cos30+0.5C
Another Solution 532
Amount of friction under block A
Sum of all vertical forces in black A
fA=NA=tan15(2cos30+0.5C)
RAcos45=2
fA=2cos30tan15+0.5Ctan15
RA=22 kN

Summation of forces on block A parallel to the 30


Sum of all horizontal forces in block A
incline
C=RAsin45
fA+2sin30=Csin60
C=22sin45
(2cos30tan15+0.5Ctan15)+1=Csin60
C=2 kN
2cos30tan15+1=Csin600.5Ctan15

(sin600.5tan15)C=2cos30tan15+1

C=[2cos30tan15+1]/[sin600.5tan15]

C=2 kN

Summation of forces on block B normal to the 45


incline
NB=2cos45+Ccos45

NB=2cos45+2cos45
Sum of all forces normal to the incline in block B
RBcos15=Ccos45+2cos45

RBcos15=2cos45+2cos45

RB=2.9282 kN

Sum of all forces parallel to the incline in block B


P+2sin45=Csin45+RBsin15

P+2sin45=2sin45+2.9282sin15

P=0.7578 kN answer MA=0

Problem 533 NB(2x)=424x+fB(2x)


A uniform bar AB, weighing 424 N, is fastened by a
frictionless pin to a block weighing 200 N as shown 2NB=424+2fB
in Fig. P-533. At the vertical wall, = 0.268 while
under the block, = 0.20. Determine the force P
2NB=424+2(0.268NB)
needed to start motion to the right.

1.464NB=424

NB=289.62 N

fB=0.268(289.62)

fB=77.62 N

FV=0

NA=fB+200+424

NA=77.62+200+424

Solution 533 NA=701.62 N

fA=0.20(701.62)

fA=140.32 N
FH=0 sin=sin2

P=fA+NB =2 answer

P=140.32+289.62

P=429.94 N answer Problem 536


in Fig. P-536, determine the minimum weight of
block B that will keep it at rest while a force P starts
blocks A up the incline surface of B. The weight of
Problem 535 A is 100 lb and the angle of friction for all surfaces
A wedge is used to split logs. If is the angle of in contact is 15.
friction between the wedge and the log, determine
the maximum angle a of the wedge so that it will
remain embedded in the log.

Solution 535

Solution 536

From the
FBD of block
FH=0 A
FV=0
P=2Rsin(+1/2)

P/=1/2Rcos(+1/2)=0

cos(+1/2)=0

+1/2=90

2+=180

=1802

sin=sin(1802)

sin=sin180cos2cos180sin2

sin=(0)(cos2)(1)(sin2)
R1cos45=100 From the FBD
of 40 kN block
R1=141.42 lb FH=0

From the FBD of block B


FH=0

R2sin15=R1sin45

R2sin15=141.42sin45

R2=386.37 lb

FV=0

WB+R1cos45=R2cos15

WB+141.42cos45=386.37cos15 R1sin80=R2sin30

WB=273.20 lb answer R1=[R2sin30]/sin80

R1=0.5077R2

Problem 537
In Fig. P-537, determine the value of P just FV=0
sufficient to start the 10 wedge under the 40-kN
block. The angle of friction is 20 for all contact R2cos30+R1cos80=40
surfaces.
R2cos30+(0.5077R2)cos80=40

0.9542R2=40

R2=41.92 kN

From the FBD of lower block


FV=0

R3cos20=R2cos30

R3cos20=41.92cos30

Solution 537 R3=38.634 kN


FH=0 R1cos20circ=R2sin10

P=R2sin30+R3sin20 R1=[R2sin10]/cos20

P=41.92sin30+38.634sin20 R1=0.1848R2

P=34.174 kN answer
FV=0

R1sin20+R2cos10=40
Problem 538
In Problem 537, determine the value of P acting to (0.1848R2)sin20+R2cos10=40
the left that is required to pull the wedge out from
under the 40-kN block. 1.048R2=40

R2=38.168 kN

From the FBD of lower block


FV=0

R3cos20=R2cos10

R3cos20=38.168cos10

R3=40 kN

Solution 538
FH=0
From the FBD of
40 kN block P=R2sin10+R3sin20
FH=0
P=38.168sin10+40sin20

P=20.308 kN answer

Problem 539
The block A in Fig. P-539 supports a load W = 100
kN and is to be raised by forcing the wedge B under
it. The angle of friction for all surfaces in contact is
f = 15. If the wedge had a weight of 40 kN, what
value of P would be required (a) to start the wedge
under the block and (b) to pull the wedge out from
under the block?
R2cos35=R1sin15+100

R2cos35=(0.5938R2)sin15+100

0.6655R2=100

R2=150.27 kN

From FBD of block B


FV=0

R3cos15=R2cos35+40

Solution 539 R3cos15=150.27cos35+40

R3=168.85 kN

FH=0

P=R2sin35+R3sin15

P=150.27sin35+168.85sin15

P=129.89 kN answer

Problem 540
As shown in Fig. P-540, two blocks each weighing
20 kN and resting on a horizontal surface, are to be
pushed apart by a 30 wedge. The angle of friction
is 15 for all contact surfaces. What value of P is
required to start movement of the blocks? How
would this answer be changed if the weight of one
Part (a): P to start the wedge under block A of the blocks were increased by 30 kN?
From the FBD of block A
FH=0

R1cos15=R2sin35

R1=0.5938R2

FV=0
From the FBD of the upper block
FV=0

P=2R2sin30

P=2(7.32)sin30

P=7.32 kN answer

Solution 540

From the FBD of 20-kN block


FH=0

R1sin15=R2cos30

R1=3.346R2

FV=0 When one block weigh 50 kN and the other is 20


kN, the first to impend when movement starts is the
R1cos15=R2sin30+20 20 kN block. Thus the reaction R2 = 7.32 kN,
similar to the above value of R2. Thus, the answer
(3.346R2)cos15=R2sin30+20 which is P = 7.32 kN will not change. See the free
body diagram below and note that the friction
2.732R2=20 reaction f3 is not equal to the maximum available
friction under the 50 kN block.
R2=7.32 kN

Problem 541
Determine the force P required to start the wedge
shown in Fig. P-541. The angle of friction for all
surfaces in contact is 15.
From the FBD of the wedge to the left
FH=0

R3cos30=R2cos15

R3cos30=115.54cos15

R3=128.87 kN

FV=0

P=R2sin15+R3sin30

P=115.54sin15+128.87sin30
Solution 541
P=94.34 kN answer
From the FBD of the block to the right
FV=0

R1cos15=R2sin15+200 Problem 542


What force P must be applied to the wedges shown
in Fig. P-542 to start them under the block? The
R1=0.2679R2+207.06
angle of friction for all contact surfaces is 10.

FH=0

R2cos15=R1sin15+50

R2cos15=(0.2679R2+207.06)sin15+50

0.8966R2=103.59

R2=115.54 kN

Solution 542

From the FBD of 1000 lb block


FV=0

2(R1cos25)=1000

R1=551.69 lb
Solution 543

From the
FBD of
the upper
From the FBD of any of the wedges
wedge
FV=0
FV=0
R2cos10=R1cos25

R2cos10=551.69cos25

R2=507.71 lb

FH=0
R2cos30=200
P=R1sin25+R2sin10
R2=230.94 kN
P=551.69sin25+507.71sin10

P=321.32 lb answer From the FBD of the lower wedge


FV=0

R3cos25=R2cos30
Problem 543
To adjust the vertical position of a column R3cos25=230.94cos30
supporting 200-kN load, two 5 wedges are used as
shown in Fig. P-543. Determine the force P R3=220.68 kN
necessary to start the wedges is the angle of friction
at all contact surfaces is 25. Neglect friction at the
FH=0
rollers.
P=R2sin30+R3sin25
P=230.94sin30+220.68sin25

P=208.73 kN answer

Problem 544
The block A in Fig. P-544 supports a load W and is
to be raised by forcing the wedge B under it. If the
angle of friction is 10 at all surfaces in contact,
determine the maximum wedge angle that will
give the wedge a mechanical advantage; i.e., make
P less than the weight W of the block.

=18080(80)

=20+

R2/sin80=P/sin

R2=[Psin80]/sin
Solution 544
[Wsin100]/sin(70)=[Wsin80]/sin(20+)
=180100(10+)
sin100sin(70)=sin80sin(20+)
=70
sin100sin(20+)=sin80sin(70)

R2/sin100=W/sin sin100(sin20cos+cos20sin)=sin80(sin70cos
cos70sin)
R2/sin100=W/sin(70)
sin100sin20cos+sin100cos20sin=sin80sin7
R2=Wsin100/sin(70) 0cossin80cos70sin

sin100cos20sin+sin80cos70sin=sin80sin70
cossin100sin20cos

(sin100cos20+sin80cos70)sin=(sin80sin70
sin100sin20)cos
Sin/cos=[sin80sin70sin100sin20]/[sin100
cos20+sin80cos70]

tan=[sin80sin70sin100sin20]/[sin100cos20
+sin80cos70]

tan=0.4663076582

=25 answer

Potrebbero piacerti anche